Eufisky - The lost book

多重积分计算的一些题

(1)设$f$在$D:x^2+y^2\leq1$上二阶连续可微,且\[\Delta f=\frac{\partial^2f}{\partial x^2}+\frac{\partial^2f}{\partial y^2}=x^2+y^2,\]求\[\iint\limits_D\left(\frac{x}{\sqrt{x^2+y^2}}\frac{\partial f}{\partial x}+ \frac{y}{\sqrt{x^2+y^2}}\frac{\partial f}{\partial y}\right)\mathrm{d}x\mathrm{d}y.\]


cool解:(Hansschwarzkopf)根据Gauss公式

\begin{align*}&\iint\limits_D\left(\frac{x}{\sqrt{x^2+y^2}}\frac{\partial f}{\partial x}+ \frac{y}{\sqrt{x^2+y^2}}\frac{\partial f}{\partial y}\right)\mathrm{d}x\mathrm{d}y \\&=\int_0^1\mathrm{d}r\int\limits_{x^2+y^2 = r^2}\left(\frac{x}{\sqrt{x^2+y^2}}\frac{\partial f}{\partial x}+ \frac{y}{\sqrt{x^2+y^2}}\frac{\partial f}{\partial y}\right)\mathrm{d}s\\ &=\int_0^1\mathrm{d}r\int\limits_{x^2+y^2 = r^2}\frac{\partial f}{\partial n}\mathrm{d}s=\int_0^1\mathrm{d}r\iint\limits_{x^2+y^2 \leqslant r^2}\Delta f\mathrm{d}x\mathrm{d}y\\&=\int_0^1\mathrm{d}r\iint\limits_{x^2+y^2 \leqslant r^2}(x^2+y^2)\mathrm{d}x\mathrm{d}y=\int_0^1\frac{\pi r^4}{2}\mathrm{d}r =\frac{\pi}{10} .\end{align*}
 
(2)设$f$在$D:x^2+y^2\leq1$上二阶连续可微,且\[\Delta f=\frac{\partial^2f}{\partial x^2}+\frac{\partial^2f}{\partial y^2}=\exp{(-x^2-y^2)},\]求

\[\iint\limits_D\left(x\frac{\partial f}{\partial x}+ y\frac{\partial f}{\partial y}\right)\mathrm{d}x\mathrm{d}y.\]


cool解:(Hansschwarzkopf)根据Gauss公式

\begin{align*}&\iint\limits_D\left(x\frac{\partial f}{\partial x}+ y\frac{\partial f}{\partial y}\right)\mathrm{d}x\mathrm{d}y\\ &=\int_0^1\mathrm{d}r\int\limits_{x^2+y^2 = r^2}\left(x\frac{\partial f}{\partial x}+y\frac{\partial f}{\partial y}\right)\mathrm{d}s\\ &=\int_0^1\mathrm{d}r\int\limits_{x^2+y^2 =r^2}r\frac{\partial f}{\partial n}\mathrm{d}s=\int_0^1\mathrm{d}r\iint\limits_{x^2+y^2 \leqslant r^2}r\Delta f\mathrm{d}x\mathrm{d}y\\ &=\int_0^1\mathrm{d}r\iint\limits_{x^2+y^2 \leqslant r^2}r\exp{(-x^2-y^2)}\mathrm{d}x\mathrm{d}y=\int_0^1\pi r(1-e^{-r^2})\mathrm{d}r =\frac{\pi }{2e} .\end{align*}

浙江大学2012年研究生入学考试高等代数试题参考解答

浙江大学2012年研究生入学考试高等代数试题参考解答
 
 
高等代数资源网
 
2013.8.2
1. 声明
您现在看到的这份文件来自http://www.52gd.org.本站原创的内容,采用创作共用组织(Creative Commons)的“公共领域”(\href{http://creativecommons.org/about/pdm}{Public Domain})许可。即放弃一切权利,全归公共领域。但涉及到其他版权人的摘录、转载、投稿、翻译等类内容不在此列。
 
本文的内容仅供学习参考之用,作者不对内容的正确性作任何承诺,作者不对因使用本文而造成的一切后果承担任何责任.
 
关于如何使用本文的建议:首先保证自己认真做了一遍题目,否则请不要查看本文.记住:
 
别人做是别人的,自己做才是自己的 .
作者水平有限,错误不可避免,欢迎您来信指出:\href{mailto:www52gdorg@163.com}{www52gdorg@163.com}.
 
2. 试题

每题15分.

 

一.设$E$是$n$阶单位矩阵,

$$M=\begin{pmatrix}0&E\\-E&0\end{pmatrix},$$

矩阵$A$满足$A^{T}MA=M.$证明$A$的行列式等于1.

 

二.设$A$是$n$阶幂等矩阵,满足

 

(1)$A=A_{1}+\cdots+A_{s};$

 

(2)$r(A)=r(A_{1})+\cdots+r(A_{s}).$

 

证明:所有的$A_{i}$都相似于一个对角阵,$A_{i}$的特征值之和等于矩阵$A_{i}$的秩.

 

三.设$\phi$是$n$维欧氏空间的正交变换,证明:$\phi$最多可以表示为$n+1$个镜面反射的复合.

 

四.设$A$是$n$阶复矩阵,证明存在常数项等于0的多项式$g(\lambda),h(\lambda)$使得$g(A)$是可以对角化的矩阵,$h(A)$是幂零矩阵,且$A=g(A)+h(A).$

 

五.设$A=\begin{pmatrix}3&2&-2\\k&-1&-k\\4&2&-3\end{pmatrix}.$(i)当$k$为何值时,存在矩阵$P$使得$P^{-1}AP$为对角矩阵?并求出这样的矩阵$P$和对角矩阵.(ii)求$k=2$时矩阵$A$的Jordan标准形.

 

六.令二次型$f(x_{1},\cdots,x_{n})=\sum_{i=1}^{m}(a_{i1}x_{1}+\cdots+a_{in}x_{n})^{2}.$

 

(i)求此二次型的方阵.

 

(ii)当$a_{ij}$均为实数时,给出此二次型为正定的条件.

 

七.设$V$和$W$是数域$K$上的线性空间,$Hom_{K}(V,W)$表示$V$到$W$的所有线性映射组成的线性空间.证明:对$f,g\in Hom_{K}(V,W),$若$Imf\cap Img=\{0\},$则$f,g$在$Hom_{K}(V,W)$中是线性无关的.

 

八.令线性空间$V=Imf\oplus W,$其中$W$是线性变换$f$的不变子空间.

 

(i)证明$W\subseteq Kerf;$

 

(ii)证明若$V$是有限维线性空间,则$W=Kerf;$

 

(iii)举例说明,当$V$是无限维的,可能有$W\subseteq Ker f,$且$W\neq Kerf.$

 

九.设$A=\begin{pmatrix}1&0&-1&2&1\\-1&1&3&-1&0\\-2&1&4&-1&3\\3&-1&-5&1&-6\end{pmatrix}.$

 

(i)求$5\times 5$阶秩为2的矩阵$M,$使得$AM=0;$

 

(ii)假如$B$是满足$AB=0$的$5\times5$阶矩阵,证明:秩$\mathrm{rank\,}(B)\leq2.$

 

十.令$T$是有限维线性空间$V$的线性变换,设$W$是$V$的$T-$不变子空间.那么$T|_{W}$的最小多项式整除$T$的最小多项式.

 

 

 

3. 参考解答

一.设$E$是$n$阶单位矩阵,

$$M=\begin{pmatrix}0&E\\-E&0\end{pmatrix},$$

矩阵$A$满足$A^{T}MA=M.$证明$A$的行列式等于1.

 

\textbf{证明:}(法1)将$A$分块为

$$A=\begin{pmatrix}A_{1}&A_{2}\\A_{3}&A_{4}\end{pmatrix}$$

由$A^{T}MA=M$有

$$\begin{aligned}-A_{2}A_{1}^{T}+A_{1}A_{2}^{T}&=&0\\-A_{2}A_{3}^{T}+A_{1}A_{4}^{T}&=&I\\\end{aligned}$$

 

若$A_{1}$可逆,由$A$的分块可得$$|A|=|A_{1}||A_{4}-A_{3}A_{1}^{-1}A_{2}|$$由上面第一式可得$$A_{2}=A_{1}A_{2}^{T}(A_{1}^{-1})^{T}$$

代入第二式可得

$$A_{4}-A_{3}A_{1}^{-1}A_{2}=(A_{1}^{-1})^{T}$$

从而可得$|A|=1$.

 

当$A_{1}$不可逆时,考虑矩阵$A_{1}+tE,$则存在无穷多$t$的值使得$A_{1}+tE$可逆(这是因为$|A_{1}+tE|$是关于$t$的一个多项式,只能有有限个根.),由前面的证明有

$$1=\begin{vmatrix}A_{1}+tE&A_{2}\\A_{3}&A_{4}\end{vmatrix}.$$

此式两边是关于$t$的多项式,且有无穷多$t$的值使得等式成立,从而等式恒成立.令$t=0$可得.

 

(法2)博士数学论坛(\url{www.math.org.cn})的mxcandy提供.

 

由$A^{T}MA=M$有

$$M(\lambda E-A)=\lambda M-MA=\lambda A^{T}MA-MA=(\lambda A^{T}-E)MA.$$

两边取行列式,由$|M|=1\neq0$得

$$|\lambda E-A|=|A||\lambda A^{T}-E|.$$

注意到$A$是$2n$阶矩阵,以及矩阵的转置行列式不变有

$$|\lambda A^{T}-E|=|E-\lambda A^{T}|=|(E-\lambda A^{T})^{T}|=|E-\lambda A|.$$

于是

$$|\lambda E-A|=|A||\lambda A^{T}-E|=|A||E-\lambda A|.$$

记$A$的特征多项式$|\lambda E-A|=f(\lambda),$则由上式有

$$f(\lambda)=|\lambda E-A|=|A|\lambda^{2n}f(\dfrac{1}{\lambda}).\eqno(*)$$

考虑$\lambda=1,$设$2n$次多项式$f(\lambda)$有分解式

$$f(\lambda)=(\lambda-1)^{m}g(\lambda),g(1)\neq0,0\leq m\leq 2n.$$

已知$A^{T}MA=M,$两边取行列式,可得$|A|^{2}=1,$从而$A$可逆,故

$$MA=(A^{T})^{-1}M,$$

由平滑性或者归纳法可得,对任意自然数$k$有

$$M(E-A)^{k}=(E-(A^{T})^{-1})^{k}M,$$

从而

$$M(E-A)^{2k}=M(E-A)^{k}(E-A)^{k}=(E-(A^{T})^{-1})^{k}M(E-A)^{k}=-(A^{T})^{-k}(E-A)^{k}M(E-A)^{k}.$$

由$M^{T}=-M$知,$(E-A)^{k}M(E-A)^{k}$反对称,注意到$|M|=1,$从而

$$\mathrm{rank\,}((E-A)^{2k})=\mathrm{rank\,}((E-A)^{k}M(E-A)^{k}).$$

由于反对称矩阵的秩为偶数,从而$\mathrm{rank\,}((E-A)^{2k})$为偶数.特别的,任取$2k\geq m,$则特征值$\lambda=1$的代数重数$m$为偶数,即

$$m=2n-\mathrm{rank\,}((E-A)^{2k})\triangleq 2p,2k\geq m,0\leq p\leq n.$$

把$f(\lambda)=(\lambda-1)^{2p}g(\lambda)$代入到(*)式,得

$$(\lambda-1)^{2p}g(\lambda)=|A|\lambda^{2n}(\dfrac{1}{\lambda}-1)^{2p}g(\dfrac{1}{\lambda}),$$

$$g(\lambda)=|A|\lambda^{2n-2k}g(\dfrac{1}{\lambda}),$$

令$\lambda=1,$并注意到$g(1)\neq0,$可得$|A|=1.$

 

注1:满足题目条件的矩阵$A$称为辛矩阵.

 

注2:由上述证明知:辛矩阵的特征多项式自反,特征值互倒成对,$\lambda=\pm1$代数重数为偶数.

 

(法3)许以超,线性代数与矩阵论(第二版).高等教育出版社.P329.

 

(法4)许以超,线性代数与矩阵论(第二版).高等教育出版社.P405.

 

(法5)高等代数中的一些问题.博士数学论坛(\url{www.math.org.cn})xida.P7.

二.设$A$是$n$阶幂等矩阵,满足

 

(1)$A=A_{1}+\cdots+A_{s};$

 

(2)$r(A)=r(A_{1})+\cdots+r(A_{s}).$

 

证明:所有的$A_{i}$都相似于一个对角阵,$A_{i}$的特征值之和等于矩阵$A_{i}$的秩.

 

\textbf{证明:}只需证明$A_{i}$是幂等矩阵.利用$n$阶矩阵$C$是幂等矩阵的充要条件为$r(C)+r(C-E)=n,$只需证明$r(A_{i}-E)=n-r(A_{i}).$利用矩阵秩的不等式

$$|r(A)-r(B)|\leq r(A\pm B)\leq r(A)+r(B)$$

以及题目条件有

$$\begin{aligned}n-r(A_{i})\leq r(A_{i}-E)&=r(A-E-(A_{1}+\cdots+A_{i-1}+A_{i+1}+\cdots+A_{s}))\\&\leq r(A-E)+r(A_{1}+\cdots+A_{i-1}+A_{i+1}+\cdots+A_{s})\\&\leq r(A-E)+r(A_{1})+\cdots+r(A_{i-1})+r(A_{i+1})+\cdots+r(A_{s})\\&=n-r(A)+r(A)-r(A_{i})\\&=n-r(A_{i}).\end{aligned}$$

从而$r(A_{i}-E)=n-r(A_{i}).$

 

三.设$\phi$是$n$维欧氏空间的正交变换,证明:$\phi$最多可以表示为$n+1$个镜面反射的复合.

 

\textbf{证明:}(法1)设$\alpha$是$n$维欧氏空间$V$中的单位向量,定义线性变换

$$\sigma(\beta)=\beta-(\beta,\alpha)\alpha,\forall\beta\in V,$$

则$\sigma$是$V$的正交变换,称为镜面反射(镜像变换).计算可得$\sigma^{2}=I$(恒等变换).

 

设$\alpha_{1},\alpha_{2}$是$n$维欧氏空间$V$中的两个长度相等的不同向量,则存在镜面反射$\sigma$使得$\sigma(\alpha_{1})=\alpha_{2}.$

实际上,令$\alpha=\dfrac{\alpha_{1}-\alpha_{2}}{|\alpha_{1}-\alpha_{2}|},$定义$\sigma(\beta)=\beta-(\beta,\alpha)\alpha,\forall\beta\in V$即可.

 

下面证明原问题.对空间的维数$n$用数学归纳法.

 

当$n=1$时,设$e_{1}$是$V$的单位向量,则$V=L(e_{1}).$由于$\phi(e_{1})\in V,$故存在实数$\lambda$使得$\phi(e_{1})=\lambda e_{1},$由$\phi$是正交变换可得

$$1=(e_{1},e_{1})=(\phi(e_{1}),\phi(e_{1}))=\lambda^{2}(e_{1},e_{1})=\lambda^{2},$$

因此$\lambda=\pm1.$令

$$\tau(\alpha)=\alpha-2(\alpha,e_{1})e_{1},\forall\alpha\in V,$$

则$\tau$是镜面反射,且当$\lambda=1$时,对$\forall\alpha\in V,$设$\alpha=ke_{1},k\in R$则

$$\phi(\alpha)=k\phi(e_{1})=ke_{1},\forall\alpha=ke_{1}=\alpha,\in V,$$

即$\phi$是恒等变换.而

$$\tau^{2}(\alpha)=k\tau^{2}(e_{1})=k\tau(-e_{1})=ke_{1}=\alpha,$$

即$\tau^{2}$也是恒等变换,从而$\phi=\tau^{2}.$而当$\lambda=-1$时,显然$\phi=\tau.$

 

假设结论对$n-1$维欧氏空间成立,对$n$维欧氏空间$V$的一正交变换$\phi,$若$\phi=I,$则对$V$的任一镜面反射$\sigma$有$\phi=I=\sigma^{2}.$若$\phi\neq I,$则存在$V$的单位向量$e$使得$\phi(e)=\eta\neq e,$由于$|\eta|=|\phi(e)|,$从而存在$V$的镜面反射$\tau$使得$$\tau(\eta)=e.$$

于是$$\tau(\phi(e))=e.$$

令$W=L(e),$由于$\tau\phi$仍为正交变换,故$W^{\bot}$是$\tau\phi$的$n-1$维不变子空间,且$\tau\phi|_{W^{\bot}}$为正交变换.由归纳假设,在$W^{\bot}$中存在单位向量$\alpha_{1},\alpha_{2},\cdots,\alpha_{k},$它们分别决定$W^{\bot}$的镜面反射$\sigma_{1},\sigma_{2},\cdots,\sigma_{k}$使得

$$\tau\phi|_{W^{\bot}}=\sigma_{1}\sigma_{2}\cdots\sigma_{k},$$

现将$\sigma_{i}$的定义扩大到$V,$即补充定义$\sigma_{i}(e)=e.$则$\sigma_{i}$即为$\alpha_{i}$决定的$V$的镜面反射.这是因为$\forall\alpha\in V,$设$\alpha=\beta_{1}+\beta_{2},\beta_{1}=ke\in W=L(e),\beta_{2}\in W^{\bot},$注意到$(\beta_{1},\alpha_{i})=0,$则

 

$$\begin{aligned}\sigma_{i}(\alpha)&=\sigma_{i}(\beta_{1})+\sigma(\beta_{2})\\&=\beta_{1}+\beta_{2}-2(\beta_{2},\alpha_{i})\alpha_{i}\\&=\alpha-2(\alpha,\alpha_{i})\alpha_{i}.\end{aligned}$$

现在显然有$\sigma_{1}\sigma_{2}\cdots\sigma_{k}(\beta_{1})=\beta_{1},$这是因为$\tau\phi(e)=e,$故$\tau\phi(\beta_{1})=\beta_{1}.$从而

 

$$\begin{aligned}\tau\phi(\alpha)&=\tau\phi(\beta_{1})+\tau\phi(\beta_{2})\\&=\beta_{1}+\sigma_{1}\sigma_{2}\cdots\sigma_{k}(\beta_{2})\\&\sigma_{1}\sigma_{2}\cdots\sigma_{k}(\beta_{1})+\sigma_{1}\sigma_{2}\cdots\sigma_{k}(\beta_{2})\\&=\sigma_{1}\sigma_{2}\cdots\sigma_{k}(\alpha)\end{aligned}$$

 

从而$\tau\phi=\sigma_{1}\sigma_{2}\cdots\sigma_{k}.$注意到$\tau^{2}=I,$有

$$\phi=\tau\sigma_{1}\sigma_{2}\cdots\sigma_{k}.$$

 

(法2)$n$阶矩阵$M=E-2\alpha\alpha^{T},$其中$\alpha$是$n$维实列向量,且$\alpha^{T}\alpha=1.$则矩阵$M$是正交矩阵,称为镜像矩阵.容易验证$M^{2}=E.$即单位矩阵是两个镜像矩阵之积.

 

设$\alpha,\beta$是两个不同的$n$维实列向量,且$|\alpha|=|\beta|,$则存在实镜像矩阵$M$使得$M\alpha=\beta.$实际上,令$\alpha=\dfrac{\alpha-\beta}{|\alpha-\beta|},M=E-2\alpha\alpha^{T}$即可.

 

可以证明欧氏空间中的线性变换$\phi$是镜面反射的充要条件是$\phi$在一组标准正交基下的矩阵为镜像矩阵.

 

这样要证明原问题,只需证明任意$n$阶实正交矩阵$A$可以分解不超过$n+1$个镜像矩阵之积即可.

 

对矩阵的阶数$n$用数学归纳法.

$n=1$时,结论显然成立.

 

假设结论对$n-1$阶矩阵成立,将$n$阶正交矩阵$A$按列分块为

$$A=(\alpha_{1},\alpha_{2},\cdots,\alpha_{n}),$$

则$|\alpha_{1}|=1,$从而存在镜像矩阵$M_{1}$使得$M_{1}\alpha_{1}=(1,0,\cdots,0)^{T},$注意到$M_{1}A$还是正交矩阵,必有

 

 

$$M_{1}A=M_{1}(\alpha_{1},\alpha_{2},\cdots,\alpha_{n})=(M_{1}\alpha_{1},M_{1}\alpha_{2},\cdots,M_{1}\alpha_{n})=\begin{pmatrix}1&0&\cdots&0\\0& &  &\\\vdots&&Q_{1}&\\0&&&\end{pmatrix}$$

 

 

容易验证$Q_{1}$也是正交矩阵,从而由归纳假设,存在$n-1$阶镜像矩阵$M_{2},\cdots,M_{k}$使得$$Q_{1}=M_{2}\cdots M_{k},$$

于是

$$A=M_{1}^{-1}\begin{pmatrix}1&0&\cdots&0\\0& &  &\\\vdots&&Q_{1}&\\0&&&\end{pmatrix}=M_{1}^{-1}\begin{pmatrix}1&\\&M_{2}\cdots M_{k}\end{pmatrix}=M_{1}^{-1}\begin{pmatrix}1&\\&M_{2}\end{pmatrix}\cdots\begin{pmatrix}1&\\&M_{k}\end{pmatrix}.$$

 

易知$\begin{pmatrix}1&\\&M_{i}\end{pmatrix}$都是镜像矩阵.

 

 

 

四.设$A$是$n$阶复矩阵,证明存在常数项等于0的多项式$g(\lambda),h(\lambda)$使得$g(A)$是可以对角化的矩阵,$h(A)$是幂零矩阵,且$A=g(A)+h(A).$

 

\textbf{证明:}等我看看能否找到一个好的方法.

 

五.设$A=\begin{pmatrix}3&2&-2\\k&-1&-k\\4&2&-3\end{pmatrix}.$(i)当$k$为何值时,存在矩阵$P$使得$P^{-1}AP$为对角矩阵?并求出这样的矩阵$P$和对角矩阵.(ii)求$k=2$时矩阵$A$的Jordan标准形.

 

\textbf{证明:}由于

$$|A-\lambda E|=\begin{vmatrix}3-\lambda&2&-2\\k&-1-\lambda&-k\\4&2&-3-\lambda\end{vmatrix}=-(\lambda+1)^{2}(\lambda-1),$$

故$A$的特征值为$$\lambda_{1}=-1(\mbox{二重}),\lambda_{2}=1.$$

 

(i)存在矩阵$P$使得$P^{-1}AP$为对角矩阵的充要条件是特征值的代数重数等于几何重数,即$r(A-\lambda_{1}E)=1,$而

$$A-\lambda_{1}E=\begin{pmatrix}4&2&-2\\k&0&-k\\4&2&-2\end{pmatrix},$$

 

从而$k=0.$

$P$可以是

$$P=\begin{pmatrix}1&1&1\\-2&0&0\\0&2&1\\\end{pmatrix},$$此时$P^{-1}AP=diag(-1,-1,1).$

 

(2)$k=2$时

 

$$\lambda E-A=\begin{pmatrix}\lambda-3&-2&2\\-2&\lambda+1&2\\-4&-2&\lambda+3\end{pmatrix}\rightarrow\begin{pmatrix}1&&\\&1&\\&&(\lambda+1)^{2}(\lambda-1)\end{pmatrix},$$

 

所以$A$的Jordan标准形为$$\begin{pmatrix}-1&1&\\&-1&\\&&1\end{pmatrix}.$$

 

六.令二次型$f(x_{1},\cdots,x_{n})=\sum_{i=1}^{m}(a_{i1}x_{1}+\cdots+a_{in}x_{n})^{2}.$

 

(i)求此二次型的方阵.

 

(ii)当$a_{ij}$均为实数时,给出此二次型为正定的条件.

 

\textbf{证明:}(i)由于

$$(a_{i1}x_{1}+\cdots+a_{in}x_{n})^{2}=(x_{1},\cdots,x_{n})\begin{pmatrix}a_{i1}\\\vdots\\a_{in}\end{pmatrix}\begin{pmatrix}a_{i1}&\cdots&a_{in}\end{pmatrix}\begin{pmatrix}x_{1}\\\vdots\\x_{n}\end{pmatrix},$$

 

\begin{align*}f(x_{1},\cdots,x_{n})&=\sum_{i=1}^{m}(a_{i1}x_{1}+\cdots+a_{in}x_{n})^{2}\\&=\sum_{i=1}^{n}(x_{1},\cdots,x_{n})\begin{pmatrix}a_{i1}\\\vdots\\a_{in}\end{pmatrix}\begin{pmatrix}a_{i1}&\cdots&a_{in}\end{pmatrix}\begin{pmatrix}x_{1}\\\vdots\\x_{n}\end{pmatrix}\\&=(x_{1},\cdots,x_{n})\begin{pmatrix}\sum_{i=1}^{n}a_{i1}^{2}&\cdots&\sum_{i=1}^{n}a_{i1}a_{in}\\\vdots&\vdots&\vdots\\\sum_{i=1}^{n}a_{in}a_{i1}&\cdots&\sum_{i=1}^{n}a_{in}^{2}\end{pmatrix}\begin{pmatrix}x_{1}\\\vdots\\x_{n}\end{pmatrix}\end{align*}.

 

若记$A=(a_{ij})_{n\times n},$则$f(x_{1},\cdots,x_{n})=(x_{1},\cdots,x_{n})(A^{T}A)\begin{pmatrix}x_{1}\\\vdots\\x_{n}\end{pmatrix}.$

 

故所求矩阵为$A^{T}A.$

 

(2)当$a_{ij}$为实数时,$A^{T}A$是半正定的,故$f(x_{1},\cdots,x_{n})=(x_{1},\cdots,x_{n})(A^{T}A)\begin{pmatrix}x_{1}\\\vdots\\x_{n}\end{pmatrix}$

正定的充要条件是$r(A^{T}A)=n.$而$r(A^{T}A)=r(A),$故原二次型正定的充要条件是$r(A)=n.$

 

七.设$V$和$W$是数域$K$上的线性空间,$Hom_{K}(V,W)$表示$V$到$W$的所有线性映射组成的线性空间.证明:对$f,g\in Hom_{K}(V,W),$若$Imf\cap Img=\{0\},$则$f,g$在$Hom_{K}(V,W)$中是线性无关的.

 

\textbf{证明:}注:这里应该假设$f\neq0,g\neq0.$否则题目无意义.

反证法.假设$f=kg,k\in K,$由于$f\neq0,$故存在$\alpha\in V,$使得$0\neq f(\alpha)\in Im f\subset W,$此时

$$o\neq \dfrac{1}{k}f(\alpha)=g(\alpha)\in Img,$$

注意到$Img$是$W$的字空间,从而$f(\alpha)\in Img,$这样$0\neq f(\alpha)\in Imf\cap Img.$这与条件矛盾.

 

八.令线性空间$V=Imf\oplus W,$其中$W$是线性变换$f$的不变子空间.

 

(i)证明$W\subseteq Kerf;$

 

(ii)证明若$V$是有限维线性空间,则$W=Kerf;$

 

(iii)举例说明,当$V$是无限维的,可能有$W\subseteq Ker f,$且$W\neq Kerf.$

 

\textbf{证明:}(i)$\forall\alpha\in W,$则由条件有

$$f(\alpha)\in Imf\cap W,$$

注意到$V=Imf\oplus W,$从而$Imf\cap W=\{0\},$故$f(\alpha)=0.$即$\alpha\in Kerf.$这就证明了$W\subseteq Kerf.$

 

(2)由(i),要证明$W=Kerf,$只需证明$dim W=dim Kerf.$而由$V=Imf\oplus W$以及维数公式$dimV=dim Imf+dim Kerf$有

$$dim W=dimV-dim Imf=dim Kerf.$$

从而结论成立.

 

(3)例:$V=P[x]$是数域$P$上关于$x$的一元多项式的全体,则$V$是无限维线性空间,$f(p(x))=p'(x)$为$V$上的求导线性变换,则

此时$Imf=V,Kerf=P,W=\{0\}.$

 

九.设$A=\begin{pmatrix}1&0&-1&2&1\\-1&1&3&-1&0\\-2&1&4&-1&3\\3&-1&-5&1&-6\end{pmatrix}.$

 

(i)求$5\times 5$阶秩为2的矩阵$M,$使得$AM=0;$

 

(ii)假如$B$是满足$AB=0$的$5\times5$阶矩阵,证明:秩$\mathrm{rank\,}(B)\leq2.$

 

\textbf{证明:}将$M$按列分块为

$$M=(m_{1},m_{2},m_{3},m_{4},m_{5}),$$

$$0=AM=A(m_{1},m_{2},m_{3},m_{4},m_{5})=(Am_{1},Am_{2},Am_{3},Am_{4},Am_{5}),$$

即$Am_{i}=0,i=1,2,3,4,5.$此即$m_{i}$是线性方程组$Ax=0$的解.

 

(i)求解$Ax=0$可得其一个基础解系为

$$\alpha_{1}=(-1,2,1,0)^{T},\alpha_{2}=(3,1,0,-2,0)^{T}.$$

故可取

$$M=(\alpha_{1},\alpha_{2},\alpha_{1},\alpha_{1},\alpha_{1}).$$

 

(ii)注意到$B$的列向量是方程组$Ax=0$的解,而方程组的任一解皆可由其基础解系线性表示,故$B$的列向量可由$\alpha_{1},\alpha_{2}$线性表示,故$r(B)\leq2.$

十.令$T$是有限维线性空间$V$的线性变换,设$W$是$V$的$T-$不变子空间.那么$T|_{W}$的最小多项式整除$T$的最小多项式.

 

\textbf{证明:}易知$W$是平凡子空间,即$W=\{0\}\mbox{或}W=V$时,结论成立.

 

下面假设$0<dimW=r<dimV=n,$取$W$的一组基$\alpha_{1},\cdots,\alpha_{r},$将其扩充为$V$的一组基$\alpha_{1},\cdots,\alpha_{r},\alpha_{r+1},\cdots,\alpha_{n},$由$W$是$T$的不变子空间,则可知$T$在上述基下的矩阵为

$$T(\alpha_{1},\cdots,\alpha_{r},\alpha_{r+1},\cdots,\alpha_{n})=(\alpha_{1},\cdots,\alpha_{r},\alpha_{r+1},\cdots,\alpha_{n})\begin{pmatrix}A_{r\times r}&B\\0&C_{(n-r)\times(n-r)}\end{pmatrix}.$$

设$T|_{W},T$的最小多项式分别为$m_{T}(x),m(x),$则

$$0=m(\begin{pmatrix}A_{r\times r}&B\\0&C_{(n-r)\times(n-r)}\end{pmatrix})=\begin{pmatrix}m(A_{r\times r})&*\\0&m(C_{(n-r)\times(n-r)})\end{pmatrix},$$

从而$m(A)=0,$即$m(x)$是$T|_{W}$的零化多项式,从而$m_{T}(x)|m(x).$

转载自:http://www.52gd.org/?p=414

北京大学数学科学学院2015年直博生摸底考试试题解答

这份试题本来已经写好答案了,但因为电脑的事,里面文件都没了。下面重新给出解答:


1.(90分) 设$y=f(x)$是$\mathbb{R}$上的$C^\infty$函数,对任意整数$k\geq0$,记$M_k=\sup_{x\in\mathbb{R}}|f^{(k)}(x)|$.设$m$和$n$为两整数, $0\leq m<n$,试分别就下列情况,给出你的结论和证明.
(1)如果$M_m$和$M_n$均有界,那么对哪些整数$k$, $M_k$有界?对哪些整数$k$, $M_k$可以无界?
(2)如果$\lim_{x\to+\infty}|f^{(m)}(x)|$存在有限极限,而$M_n$有界,则对哪些自然数$k$,极限$\lim_{x\to+\infty}|f^{(k)}(x)|$也存在极限?
(3)如果$\lim_{x\to+\infty}|f^{(m)}(x)|$和$\lim_{x\to+\infty}|f^{(n)}(x)|$都存在有限极限,则对哪些自然数$k$,极限$\lim_{x\to+\infty}|f^{(k)}(x)|$也存在极限?
 

2.(30分) 判断级数$\sum\nolimits_{n = 2}^{ + \infty } {\frac{{{{\left( { - 1} \right)}^n}}}{{\sqrt n  + {{\left( { - 1} \right)}^{\left[ {\sqrt n } \right]}}}}}$的敛散性,其中$[x]$表示$x$的取整.

enlightened证:\begin{align*}\sum\limits_{n = 2}^{ + \infty } {\frac{{{{\left( { - 1} \right)}^n}}}{{\sqrt n  + {{\left( { - 1} \right)}^{\left[ {\sqrt n } \right]}}}}}  &= \sum\limits_{n = 2}^{ + \infty } {\frac{{{{\left( { - 1} \right)}^n}\left( {\sqrt n  - {{\left( { - 1} \right)}^{\left[ {\sqrt n } \right]}}} \right)}}{{n - 1}}} \\&= \sum\limits_{n = 2}^{ + \infty } {\frac{{{{\left( { - 1} \right)}^n}\sqrt n }}{{n - 1}}}  - \sum\limits_{n = 2}^{ + \infty } {\frac{{{{\left( { - 1} \right)}^{n + \left[ {\sqrt n } \right]}}}}{{n - 1}}}.\end{align*}

由Leibniz判别法知,级数\[\sum\limits_{n = 2}^{ + \infty } {\frac{{{{\left( { - 1} \right)}^n}\sqrt n }}{{n - 1}}}  = \sum\limits_{n = 2}^{ + \infty } {\frac{{{{\left( { - 1} \right)}^n}}}{{\sqrt n  - \frac{1}{{\sqrt n }}}}} \]收敛.
当$k \le \sqrt n  < k + 1$,即${k^2} \le n < {\left( {k + 1} \right)^2}$时, ${\left[ {\sqrt n } \right]}=k$,则
\begin{align*}&\sum\limits_{n = 2}^{ + \infty } {\frac{{{{\left( { - 1} \right)}^{n + \left[ {\sqrt n } \right]}}}}{{n - 1}}}  =  - \frac{1}{2} + \sum\limits_{k = 2}^{ + \infty } {\sum\limits_{n = {k^2}}^{{k^2} + 2k} {\frac{{{{\left( { - 1} \right)}^{n + k}}}}{{n - 1}}} }  =  - \frac{1}{2} + \sum\limits_{k = 2}^{ + \infty } {{{\left( { - 1} \right)}^k}\sum\limits_{n = {k^2}}^{{k^2} + 2k} {\frac{{{{\left( { - 1} \right)}^n}}}{{n - 1}}} } \\&=  - \frac{1}{2} + \sum\limits_{k = 2}^{ + \infty } {{{\left( { - 1} \right)}^k}\sum\limits_{n = {k^2}}^{{k^2} + 2k} {{{\left( { - 1} \right)}^{{k^2}}}\left[ {\left( {\frac{1}{{{k^2} - 1}} + \frac{1}{{{k^2} + 1}} +  \cdots  + \frac{1}{{{k^2} + 2k - 1}}} \right)} \right.} } \\&\left. { - \left( {\frac{1}{{{k^2}}} + \frac{1}{{{k^2} + 2}} +  \cdots  + \frac{1}{{{k^2} + 2k - 2}}} \right)} \right] =  - \frac{1}{2} + \sum\limits_{k = 2}^{ + \infty } {\left[ {\left( {\frac{1}{{{k^2} - 1}} + \frac{1}{{{k^2} + 1}} +  \cdots  + \frac{1}{{{k^2} + 2k - 1}}} \right)} \right.} \\&\left. { - \left( {\frac{1}{{{k^2}}} + \frac{1}{{{k^2} + 2}} +  \cdots  + \frac{1}{{{k^2} + 2k - 2}}} \right)} \right] \le  - \frac{1}{2} + \sum\limits_{k = 2}^{ + \infty } {\left[ {\left( {\frac{{k + 1}}{{{k^2} - 1}} - \frac{k}{{{k^2} + 2k - 2}}} \right)} \right.} \\&\le  - \frac{1}{2} + \sum\limits_{k = 2}^{ + \infty } {\left[ {\left( {\frac{1}{{k - 1}} - \frac{1}{{k + 2}}} \right)} \right.}  =  - \frac{1}{2} + 1 + \frac{1}{2} + \frac{1}{3} = \frac{4}{3}\end{align*}
\begin{align*}{a_n} &= \left( {\frac{1}{{{k^2} - 1}} + \frac{1}{{{k^2} + 1}} +  \cdots  + \frac{1}{{{k^2} + 2k - 1}}} \right) - \left( {\frac{1}{{{k^2}}} + \frac{1}{{{k^2} + 2}} +  \cdots  + \frac{1}{{{k^2} + 2k - 2}}} \right)\\&= \left( {\frac{1}{{{k^2} - 1}} - \frac{1}{{{k^2}}}} \right) + \left( {\frac{1}{{{k^2} + 1}} - \frac{1}{{{k^2} + 2}}} \right) +  \cdots  + \left( {\frac{1}{{{k^2} + 2k - 3}} - \frac{1}{{{k^2} + 2k - 2}}} \right) + \frac{1}{{{k^2} + 2k - 1}}\\&\ge \frac{1}{{{k^2} + 2k - 1}} > 0.\end{align*}
故\[\sum\limits_{n = 2}^{ + \infty } {\frac{{{{\left( { - 1} \right)}^{n + \left[ {\sqrt n } \right]}}}}{{n - 1}}} \]收敛,从而数列$\sum\nolimits_{n = 2}^{ + \infty } {\frac{{{{\left( { - 1} \right)}^n}}}{{\sqrt n  + {{\left( { - 1} \right)}^{\left[ {\sqrt n } \right]}}}}}$亦收敛.laugh

3.(30分) 证明\[\int_0^1 {\int_0^1 {\frac{1}{{{{\left( {xy} \right)}^{xy}}}}dxdy} }  = \int_0^1 {\frac{1}{{{x^x}}}dx}  = \sum\limits_{n = 1}^{ + \infty } {\frac{1}{{{n^n}}}} .\]

enlightened证:令$u = xy,v = x$,则$x=v,y=\frac uv$.由$0\leq x,y\leq 1$可知$0\leq u\leq v,0\leq v\leq 1$,

\begin{align*}\left| {\frac{{\partial \left( {x,y} \right)}}{{\partial \left( {u,v} \right)}}} \right| = \left| {\begin{array}{*{20}{c}}0&1\\{\frac{1}{v}}&{ - \frac{u}{{{v^2}}}}\end{array}} \right| = - \frac{1}{v}\,,\end{align*}
那么有
\begin{align*}&\int_0^1 {\int_0^1 {\frac{1}{{{{\left( {xy} \right)}^{xy}}}}dxdy} } = \int_0^1 {dv} \int_0^v {\frac{1}{{{u^u}v}}du} \\&= \int_0^1 {du} \int_u^1 {\frac{1}{{{u^u}v}}dv} = \int_0^1 {\frac{{ - \ln u}}{{{u^u}}}du} \\&= \int_0^1 {\frac{{ - \ln u - 1}}{{{u^u}}}du} + \int_0^1 {\frac{1}{{{u^u}}}du} \\&= \left[ {\frac{1}{{{u^u}}}} \right]_0^1 + \int_0^1 {\frac{1}{{{u^u}}}du} = \int_0^1 {\frac{1}{{{x^x}}}dx}.\end{align*}
\begin{align*}&\int_0^1 {\frac{1}{{{x^x}}}dx} = \int_0^1 {{e^{ - x\ln x}}dx} = \int_0^1 {{e^{ - x\ln x}}dx} \\&= \int_0^1 {\sum\limits_{n = 0}^{+\infty} {\frac{{{{\left( { - x\ln x} \right)}^n}}}{{n!}}} dx} = \sum\limits_{n = 0}^{+\infty} {\frac{1}{{n!}}\int_0^1 {{{\left( { - x\ln x} \right)}^n}dx} }.\end{align*}
令$t=-(n+1)\ln x$,有
\begin{align*}\int_0^1 {{{\left( { - x\ln x} \right)}^n}dx} &= \frac{1}{{{{\left( {n + 1} \right)}^{n + 1}}}}\int_0^{ + +\infty } {{t^n}{e^{ - t}}dt} \\&= \frac{{\Gamma \left( {n + 1} \right)}}{{{{\left( {n + 1} \right)}^{n + 1}}}} = \frac{{n!}}{{{{\left( {n + 1} \right)}^{n + 1}}}}.\end{align*}
因此有
\begin{align*}\int_0^1 {\frac{1}{{{x^x}}}dx} = \sum\limits_{n = 0}^{ + \infty } {\frac{1}{{n!}}\int_0^1 {{{\left( { - x\ln x} \right)}^n}dx} } = \sum\limits_{n = 0}^{ + \infty } {\frac{1}{{{{\left( {n + 1} \right)}^{n + 1}}}}} =\sum\limits_{n = 1}^{ + \infty } {\frac{1}{{{n^n}}}} .\end{align*}laugh

4.(25分) 设$A$是一个$n$阶方阵,且$n\geq3$.$A^\ast$是$A$的伴随矩阵(即$A$的代数余子式所组成的矩阵).试证明,若${(A^\ast)}^\ast\neq O$ (零矩阵),则$A$可逆,且此时${(A^\ast)}^\ast$是$A$的一个纯量倍.

enlightened证:由于$A$可逆时,$A^\ast$必可逆,从而${(A^\ast)}^\ast$亦可逆;当$A$不可逆时,$A^\ast$的秩不大于$1$,从而${(A^\ast)}^\ast$必为零矩阵.

 

由此可知,当${(A^\ast)}^\ast\neq O$ 时,$A$可逆.再由\[A{A^ * } = \left| A \right|{I_n} \Rightarrow \left| {{A^ * }} \right| = {\left| A \right|^{n - 1}},{\left( {{A^ * }} \right)^{ - 1}} = \frac{1}{{\left| A \right|}}A\]及\[{A^ * }{\left( {{A^ * }} \right)^*} = \left| {{A^ * }} \right|{I_n}\]可知

\[{\left( {{A^ * }} \right)^*} = \left| {{A^ * }} \right|{\left( {{A^ * }} \right)^{ - 1}} = {\left| A \right|^{n - 1}} \cdot \frac{1}{{\left| A \right|}}A = {\left| A \right|^{n - 2}}A.\]由于${\left| A \right|^{n - 2}}$是个定值,我们得知${(A^\ast)}^\ast$是$A$的一个纯量倍.laugh


5.(25分) 设$A$是一个3阶实方阵,考虑$A$所定义的线性变换$\mathbb{R}^3\to\mathbb{R}^3,\alpha\to A\alpha$ ( $\alpha$是列向量).试证明:若$AA'=A'A$ (其中$A'$是指$A$的转置矩阵),则上述线性变换必有一个2维不变子空间.

 


6.(25分) 设$A$和$B$是复数域$\mathbb{C}$上的两个$n$阶方阵,并且$A$有$n$个特征值$1,2,\cdots,n$, $B$也有$n$个特征值$\sqrt{p_1},\cdots,\sqrt{p_n}$,其中$p_1,\cdots,p_n$是前$n$个素数(比如$p_1=2,p_2=3$等).试证明: $M_n(\mathbb{C})$上的线性变换$X\to AXB$是可以对角化的.

 

7.(25分) 设\[A = \left( {\begin{array}{*{20}{c}}{ - 2}&1&3\\{ - 2}&1&2\\{ - 1}&1&2\end{array}} \right),\]

试找出两个没有常数项的多项式$f(x)$和$\varphi(x)$,使得下列三个条件同时成立:
1). $f(A)$可对角化.    2). $\varphi (A)$是幂零矩阵.    3). $A=f(A)+\varphi (A)$.
enlightened解:\[f\left( \lambda \right) = \left| {\lambda {I_n} - A} \right| = \left( {\lambda + 1} \right){\left( {\lambda - 1} \right)^2}.\]

由Cayley---Hamilton定理可知,\[f\left( A \right) = \left( {A + 1} \right){\left( {A - 1} \right)^2} = {A^3} - {A^2} - A + {I_n} = 0.\]

我们有

\[A = {A^3} + \left( {{A^2} - {A^4}} \right).\]取\[f\left( x \right) = {x^3},\varphi \left( x \right) = {x^2} - {x^4}\]即可.laugh


8.几何部分共5道小题,每小题10分。

(1)三维欧氏空间中取定直角坐标系。有一直线$l$过点$(1,0,0)$且方向向量为$(0,1,1)$。$l$绕$z$轴旋转生成一个二次曲面$S$。试写出此二次曲面的代数方程(形如$f(x,y,z)=0$).
(2)设有一固定平面$\Sigma$,具有以下性质:上述直线$l$在绕$z$轴旋转过程中总是与$\Sigma$相交。考虑与$\Sigma$平行的平面族$\Sigma_t,t\in\mathbb{R},\Sigma_0=\Sigma$。试证明$\Sigma_t\cap S$总是椭圆.
(3)试证明$t$值变化过程中,上述各椭圆的中心总落在一条过原点的空间定直线$L$上.
(4)固定$L$上任一点$p$,试证明:由$p$向曲面$S$作的各条切线的切点都落在一条椭圆$\Gamma_p$上,且椭圆$\Gamma_p$所在平面是$\Sigma_t$之一.
(5)$S$把它在空间的补集分成内外两个连通分支,其中外部区域不包含原点。取上一小题所述椭圆$\Gamma$所在平面落在$S$外部的一点$\hat p$。试证明:从$\hat p$向$S$所作的各条切线之切点落在一条双曲线$\hat \Gamma$上,且$\hat \Gamma$所在平面过$p$点.
enlightened证:(1)记$A(1,0,0)$,设直线上有一点$B(x,y,z)$,则$\overrightarrow {AB} = \left( {x - 1,y,z} \right)=t(0,1,1)$,则$B$为$(1,t,t)$.对于给定$z=t$,其绕$z$轴旋转形成的图形为\[{x^2} + {y^2} = {t^2} + 1 = {z^2} + 1,\]故该二次曲面方程为\[{x^2} + {y^2} - {z^2} - 1 = 0.\]

(2)设固定平面$\Sigma$的方程为$Ax+By+Cz-a_t=0$

 

  • 若$A^2+B^2=0$即$A=B=0$时,方程退化成

$z = \frac{{{a_t}}}{C}$,\[\left\{ \begin{array}{l}{x^2} + {y^2} - {z^2} = 1\\z = \frac{{{a_t}}}{C}\end{array} \right. \Rightarrow \frac{{{x^2}}}{{\frac{{a_t^2}}{{{C^2}}} + 1}} + \frac{{{y^2}}}{{\frac{{a_t^2}}{{{C^2}}} + 1}} = 1,\]

可知此时$\Sigma_t\cap S$为圆,当然可以看成是椭圆.

  • 若$A^2+B^2\neq0$时,作坐标系旋转

\[\left\{ \begin{array}{l}{x_1} = \frac{B}{{\sqrt {{A^2} + {B^2}} }}x - \frac{A}{{\sqrt {{A^2} + {B^2}} }}y\\{y_1} = - \frac{{AC}}{{\sqrt {\left( {{A^2} + {B^2}} \right)\left( {{A^2} + {B^2} + {C^2}} \right)} }}x - \frac{{BC}}{{\sqrt {\left( {{A^2} + {B^2}} \right)\left( {{A^2} + {B^2} + {C^2}} \right)} }}y + \frac{{{A^2} + {B^2}}}{{\sqrt {\left( {{A^2} + {B^2}} \right)\left( {{A^2} + {B^2} + {C^2}} \right)} }}z\\{z_1} = \frac{A}{{\sqrt {{A^2} + {B^2} + {C^2}} }}x + \frac{B}{{\sqrt {{A^2} + {B^2} + {C^2}} }}y + \frac{C}{{\sqrt {{A^2} + {B^2} + {C^2}} }}z\end{array} \right.\]

\[\left\{ \begin{array}{l}x = \frac{B}{{\sqrt {{A^2} + {B^2}} }}{x_1} - \frac{{AC}}{{\sqrt {\left( {{A^2} + {B^2}} \right)\left( {{A^2} + {B^2} + {C^2}} \right)} }}{y_1} + \frac{A}{{\sqrt {{A^2} + {B^2} + {C^2}} }}{z_1}\\y = - \frac{A}{{\sqrt {{A^2} + {B^2}} }}{x_1} - \frac{{BC}}{{\sqrt {\left( {{A^2} + {B^2}} \right)\left( {{A^2} + {B^2} + {C^2}} \right)} }}{y_1} + \frac{B}{{\sqrt {{A^2} + {B^2} + {C^2}} }}{z_1}\\z = \frac{{\sqrt {{A^2} + {B^2}} }}{{\sqrt {{A^2} + {B^2} + {C^2}} }}{y_1} + \frac{C}{{\sqrt {{A^2} + {B^2} + {C^2}} }}{z_1}\end{array} \right.,\]

该平面方程化为${z_1} = \frac{{{a_t}}}{{\sqrt {{A^2} + {B^2} + {C^2}} }}$, $S$化为

\[x_1^2 + \frac{{{C^2} - {A^2} - {B^2}}}{{{A^2} + {B^2} + {C^2}}}y_1^2 - \frac{{{C^2} - {A^2} - {B^2}}}{{{A^2} + {B^2} + {C^2}}}z_1^2 - \frac{{4C\sqrt {{A^2} + {B^2}} }}{{{A^2} + {B^2} + {C^2}}} - 1 = 0,\]

则截面方程为

\[x_1^2 + \frac{{{C^2} - {A^2} - {B^2}}}{{{A^2} + {B^2} + {C^2}}}y_1^2 - \frac{{{C^2} - {A^2} - {B^2}}}{{{{\left( {{A^2} + {B^2} + {C^2}} \right)}^2}}}a_t^2 - \frac{{4C\sqrt {{A^2} + {B^2}} }}{{{A^2} + {B^2} + {C^2}}} - 1 = 0.\]显然$\Sigma_t\cap S$为椭圆.

 

综上可知, $\Sigma_t\cap S$总是椭圆.

(3)由(2)可知,在$x_1y_1z_1$坐标系中,椭圆的中心为

\[\left( {0,0,\frac{{{a_t}}}{{\sqrt {{A^2} + {B^2} + {C^2}} }}} \right),\]即中心在$z_1$轴上,\[\left\{ \begin{array}{l}{x_1} = \frac{B}{{\sqrt {{A^2} + {B^2}} }}x - \frac{A}{{\sqrt {{A^2} + {B^2}} }}y = 0\\{y_1} = - \frac{{AC}}{{\sqrt {\left( {{A^2} + {B^2}} \right)\left( {{A^2} + {B^2} + {C^2}} \right)} }}x - \frac{{BC}}{{\sqrt {\left( {{A^2} + {B^2}} \right)\left( {{A^2} + {B^2} + {C^2}} \right)} }}y + \frac{{{A^2} + {B^2}}}{{\sqrt {\left( {{A^2} + {B^2}} \right)\left( {{A^2} + {B^2} + {C^2}} \right)} }}z = 0\end{array} \right.,\]从而在$xyz$坐标系中,椭圆中心落在过原点的定直线

\[L: \left\{ \begin{array}{l}Bx - Ay = 0\\- ACx - BCy + \left( {{A^2} + {B^2}} \right)z = 0\end{array} \right.\]

上.

(4)设\[S: x_1^2 + \frac{{{C^2} - {A^2} - {B^2}}}{{{A^2} + {B^2} + {C^2}}}y_1^2 - \frac{{{C^2} - {A^2} - {B^2}}}{{{A^2} + {B^2} + {C^2}}}z_1^2 - \frac{{4C\sqrt {{A^2} + {B^2}} }}{{{A^2} + {B^2} + {C^2}}} - 1 = 0,\]上一点$P\left( {{x_{1,0}},{y_{1,0}},{z_{1,0}}} \right)$,则曲面$S$在$P$点处的切面为

\[\left( {{x_1} - {x_{1,0}}} \right) \cdot \left( {2{x_{1,0}}} \right) + \left( {{y_1} - {y_{1,0}}} \right) \cdot \left( {2\frac{{{C^2} - {A^2} - {B^2}}}{{{A^2} + {B^2} + {C^2}}}{y_{1,0}}} \right) + \left( {{z_1} - {z_{1,0}}} \right) \cdot \left( { - 2\frac{{{C^2} - {A^2} - {B^2}}}{{{A^2} + {B^2} + {C^2}}}{z_{1,0}}} \right) = 0,\]记坐标系$xyz$中,直线$L$上任一点$p$在$x_1,y_1,z_1$中的坐标为$p_1(0,0,m)$,则$p_1$满足切面方程,即\[\left( {0 - {x_{1,0}}} \right) \cdot \left( {2{x_{1,0}}} \right) + \left( {0 - {y_{1,0}}} \right) \cdot \left( {2\frac{{{C^2} - {A^2} - {B^2}}}{{{A^2} + {B^2} + {C^2}}}{y_{1,0}}} \right) + \left( {m - {z_{1,0}}} \right) \cdot \left( { - 2\frac{{{C^2} - {A^2} - {B^2}}}{{{A^2} + {B^2} + {C^2}}}{z_{1,0}}} \right) = 0,\]\[ - 2m\frac{{{C^2} - {A^2} - {B^2}}}{{{A^2} + {B^2} + {C^2}}}{z_{1,0}} - \frac{{8C\sqrt {{A^2} + {B^2}} }}{{{A^2} + {B^2} + {C^2}}} - 2 = 0 \Rightarrow {z_{1,0}} = \frac{{\left( {{A^2} + {B^2} + {C^2}} \right) + 4C\sqrt {{A^2} + {B^2}} }}{{m\left( {{A^2} + {B^2} - {C^2}} \right)}},\]故此时的$P$的$z_1$坐标为定值, 在$x_1,y_1,z_1$中$P$形成的轨迹为椭圆,且对应在$x,y,z$中$\Sigma_t\cap S$的一个椭圆.

(5)设$x_1y_1z_1$坐标系中, $\Gamma_p: \left\{ \begin{array}{l}{z_1} = \frac{{\left( {{A^2} + {B^2} + {C^2}} \right) + 4C\sqrt {{A^2} + {B^2}} }}{{m\left( {{A^2} + {B^2} - {C^2}} \right)}}\\x_1^2 + \frac{{{C^2} - {A^2} - {B^2}}}{{{A^2} + {B^2} + {C^2}}}y_1^2 - \frac{{{C^2} - {A^2} - {B^2}}}{{{A^2} + {B^2} + {C^2}}}z_1^2 - \frac{{4C\sqrt {{A^2} + {B^2}} }}{{{A^2} + {B^2} + {C^2}}} - 1 = 0\end{array} \right.$所在平面落在$S$外部的一点$\hat p_1$的坐标为$\left( {{x_{1,1}},{y_{1,1}},{z_{1,1}}} \right)$, $\hat p_1$满足(4)中$P$点处的切面方程,即\[\left( {{x_{1,1}} - {x_{1,0}}} \right) \cdot \left( {2{x_{1,0}}} \right) + \left( {{y_{1,1}} - {y_{1,0}}} \right) \cdot \left( {2\frac{{{C^2} - {A^2} - {B^2}}}{{{A^2} + {B^2} + {C^2}}}{y_{1,0}}} \right) + \left( {{z_{1,1}} - {z_{1,0}}} \right) \cdot \left( { - 2\frac{{{C^2} - {A^2} - {B^2}}}{{{A^2} + {B^2} + {C^2}}}{z_{1,0}}} \right) = 0,\]

 

则有

\[2{x_{1,1}}{x_{1,0}} + 2\frac{{{C^2} - {A^2} - {B^2}}}{{{A^2} + {B^2} + {C^2}}}{y_{1,1}}{y_{1,0}} - \frac{{8C\sqrt {{A^2} + {B^2}} }}{{{A^2} + {B^2} + {C^2}}} - 2 + \frac{{2\left( {{A^2} + {B^2} + {C^2}} \right) + 8C\sqrt {{A^2} + {B^2}} }}{{m\left( {{A^2} + {B^2} - {C^2}} \right)}}{z_{1,0}} = 0.\]显然其经过$p_1$.进一步地,经过与之前类似的坐标轴旋转,我们知道$P$的轨迹落在一条双曲线上.


PS:这份试卷是考完后的第一天根据好友同学提供的资料进行整理的,感谢他们的辛劳,同时也祝贺他们在昨天下午清华的初试中获得成功。

 

一个很好的积分题

背景是这个:

 

 

 

 

 

 

 

 

 

 

 

 

 

 

然后郝XX跟我说了下他的求法,还是挺有意思的!


\[\int_0^{2\pi } {{e^{\cos x}}\cos \left( {\sin x} \right)\cos nxdx} .\]


由Euler公式,我们有

\begin{align*}&{e^{y\cos x}}\cos \left( {y\sin x} \right) + i{e^{y\cos x}}\sin \left( {y\sin x} \right) = {e^{y\cos x}} \cdot {e^{iy\sin x}}\\=& {e^{y\cos x + iy\sin x}} = {e^{y\left( {\cos x + i\sin x} \right)}} = \sum\limits_{n = 0}^{ + \infty } {\frac{{{y^n}}}{{n!}}{{\left( {\cos x + i\sin x} \right)}^n}} \\=& \sum\limits_{n = 0}^{ + \infty } {\frac{{{y^n}}}{{n!}}{{\left( {\cos x + i\sin x} \right)}^n}} = \sum\limits_{n = 0}^{ + \infty } {\frac{{{y^n}}}{{n!}}\left( {\cos nx + i\sin nx} \right)} \\= &\sum\limits_{n = 0}^{ + \infty } {\frac{{{y^n}\cos nx}}{{n!}}} + i\sum\limits_{n = 0}^{ + \infty } {\frac{{{y^n}\sin nx}}{{n!}}} .\end{align*}

因此有

\[{e^{y\cos x}}\cos \left( {y\sin x} \right) = \sum\limits_{n = 0}^{ + \infty } {\frac{{{y^n}\cos nx}}{{n!}}} ,{e^{y\cos x}}\sin \left( {y\sin x} \right) = \sum\limits_{n = 0}^{ + \infty } {\frac{{{y^n}\sin nx}}{{n!}}} .\]

在第一个式子中令$y=1$,我们有

\[{e^{\cos x}}\cos \left( {\sin x} \right) = \sum\limits_{n = 0}^{ + \infty } {\frac{{\cos nx}}{{n!}}} .\]

因此我们有

\begin{align*}&\int_0^{2\pi } {{e^{\cos x}}\cos \left( {\sin x} \right)\cos nxdx} = \int_0^{2\pi } {\left( {\cos nx \cdot \sum\limits_{n = 0}^{ + \infty } {\frac{{\cos nx}}{{n!}}} } \right)dx} \\=& \int_0^{2\pi } {\left( {\cos nx \cdot \frac{{\cos nx}}{{n!}}} \right)dx} = \left\{ \begin{array}{l}\frac{\pi }{{n!}},n \ge 1\\2\pi ,n = 0\end{array} \right..\end{align*}


而对于无穷乘积\[\prod\limits_{n = 3}^\infty  {\cos \frac{\pi }{{n!}}}  \approx 0.858314.\]也就是管理员一分钟都不能禁他!!!

事实上,我们有

\begin{align*}&\int_0^\pi  {{e^{p\cos x}}\cos \left( {p\sin x} \right)\cos qxdx}  = \int_0^\pi  {{e^{p\cos x}}\cos qx{\mathop{\rm Re}\nolimits} \left( {{e^{ip\sin x}}} \right)dx} \\= &\int_0^\pi  {\cos qx{\mathop{\rm Re}\nolimits} \left( {{e^{ip\sin x + p\cos x}}} \right)dx}  = \int_0^\pi  {\cos qx{\mathop{\rm Re}\nolimits} \left( {{e^{p{e^{ix}}}}} \right)dx} \\= &\int_0^\pi  {\cos qx{\mathop{\rm Re}\nolimits} \left( {\sum\limits_{k = 0}^\infty  {\frac{{{p^k}{e^{ikx}}}}{{k!}}} } \right)dx}  = \sum\limits_{k = 0}^\infty  {\frac{{{p^k}}}{{k!}}\int_0^\pi  {\cos qx\cos kxdx} } \\= &\frac{1}{2}\sum\limits_{k = 0}^\infty  {\frac{{{p^k}}}{{k!}}\left[ {\frac{{\sin \left( {k - q} \right)x}}{{k - q}} - \frac{{\sin \left( {k + q} \right)x}}{{k + q}}} \right]} _0^\pi  = \frac{\pi }{2}\frac{{{p^q}}}{{q!}}.\end{align*}

 

华东师范大学数学系精品课程主页

华东师范大学数学系精品课程主页http://math.ecnu.edu.cn/jpkc/

 

几个逼格稍高的积分级数题

先是证明两个积分成立:

\begin{align}\int_0^{ + \infty } {\frac{{\sin nx}}{{x + \frac{1}{{x + \frac{2}{{x + \frac{3}{{x +  \cdots }}}}}}}}dx}  &= \frac{{\sqrt {\frac{\pi }{2}} }}{{n + \frac{1}{{n + \frac{2}{{n + \frac{3}{{n +  \cdots }}}}}}}}\\\int_0^{ + \infty } {\frac{{\sin \frac{{n\pi x}}{2}}}{{x + \frac{{{1^2}}}{{x + \frac{{{2^2}}}{{x + \frac{{{3^2}}}{{x +  \cdots }}}}}}}}dx}  &= \frac{1}{{n + \frac{{{1^2}}}{{n + \frac{{{2^2}}}{{n + \frac{{{3^2}}}{{n +  \cdots }}}}}}}}.\end{align}

接着是两个级数题,判断它们是否成立:

\begin{align}\sum\limits_{n = 0}^{ + \infty } {\left[ {\left( {1 + \frac{1}{3} + \frac{1}{5} +  \cdots  + \frac{1}{{2n + 1}}} \right) \cdot \frac{1}{{{5^n}\left( {2n + 1} \right)}}} \right]}  &= \frac{{{\pi ^2}}}{{4\sqrt 5 }} - \frac{{\sqrt 5 }}{{24}}{\left( {\ln \left( {2 + \sqrt 5 } \right)} \right)^2}\\\sum\limits_{n = 0}^{ + \infty } {\left[ {\left( {1 + \frac{1}{3} + \frac{1}{5} +  \cdots  + \frac{1}{{2n + 1}}} \right) \cdot \frac{1}{{{9^n}\left( {2n + 1} \right)}}} \right]}  &= \frac{{{\pi ^2}}}{8} - \frac{3}{8}{\left( {\ln 2} \right)^2}\end{align}

2015年丘赛分析组个人赛试题

  1. Let $f_n\in L^2(R)$ be a sequence of measurable functions over the line, $f_n\rightarrow f$ almost everywhere. Let $||f_n||_{L^2}\rightarrow||f||_{L^2}$, prove that $||f_n-f||_{L^2}\rightarrow 0$.

    enlightenedProof.(Weingarten) $L^2(\mathbf R)$ is a Hilbert space, and $\|f_n\|_{L^2}\to\|f\|_{L^2}$ as $n\to\infty$ , so we only need to prove $f_n\in f$ weakly in $L^2(\mathbf R)$, that is, for each $g\in L^2(\mathbf R)$, there holds

    \[\lim_{n\to\infty}\int_{\mathbf R}f_ng=\int_{\mathbf R} fg.\]

    For each $\epsilon>0$, there exists $R>0$, such that \[\int_{|x|>R}|g|^2<\epsilon^2,\]

    and by the absolute continuity of integration of $g$, there exists a positive $\delta$, such that: for any Lebesgue measurable subset $E$ of $\mathbf R$ with $m(E)<\delta$, there holds \[\int_E|g|^2<\epsilon^2.\]

    By the Egoroff's thoerem, there exists a subset $E_\delta$ of $(-R,R)$ with $m((-R,R)\setminus E_\delta)<\delta$, such that the convergence $\lim\limits_{n\to\infty}f_n=f$ is uniform on the $E_\delta$, so there exists $N\in\mathbf Z_+$, such that \[(\forall n>N,x\in E_\delta),(|f_n-f|<\epsilon/\sqrt{2R}).\]

    Assume $M=\|g\|_{L^2}+\|f\|_{L^2}+\sup\limits_n\|f_n\|_{L^2}$, hence $\forall n>N$, we have the following estimations

    \begin{align*}\int_{\mathbf R}|f_n-f|\cdot|g|&=\left(\int_{E_\delta}+\int_{(-R,R)\setminus E_\delta}+\int_{|x|>R}\right)|f_n-f|\cdot|g|\\&\leq\sqrt{\int_{E_\delta}|f_n-f|^2\cdot\int_{E_\delta}|g|^2}+\sqrt{\int_{(-R,R)\setminus E_\delta}|f_n-f|^2\cdot\int_{(-R,R)\setminus E_\delta}|g|^2}\\&+\sqrt{\int_{|x|>R}|f_n-f|^2\cdot\int_{|x|>R}|g|^2}\\&\leq M\epsilon+2\sqrt{2}M\epsilon.\end{align*}

    the proof is finished.


     

  2. Let $f$ be a continuous function on $[a,b]$, define $M_n=\int ^b_a f(x)x^n{\rm d}x$. Suppose that $M_n=0$ for all $n$, show that $f(x)=0$ for all $x$.
  3. Determine all entire functions $f$ that satisfying the inequality $$|f(z)|\leq|z|^2|{\rm Im}(z)|^2$$ for $z$ sufficlently large.
  4. Describe all holomorphic functions over the unit disk $D=\{z||z|\leq 1\}$ which maps the boundary of the disk into the boundary of the disk.
  5. Let $T:H_1\rightarrow H_2, Q:H_2\rightarrow H_1$ be bounded linear operators of Hilbert spaces $H_1,\ H_2$. Let $QT={\rm Id}-S_1,TQ={\rm Id}-S_2$ where $S_1$ and $S_2$ are compact operators. Prove ${\rm Ker}T=\{v\in H_1,Tv=0\},{\rm Coker}T=H_2/\overline{{\rm Im}T}$, where ${\rm Im}T=\{Tv\in H_2,v\in H_1\}$ are finite dimensional and ${\rm Im}(T)$ is closed in $H_2$.
  6. Let $H_1$ be the Sobolev space on the unit interval $[0,1]$, i.e. the Hilbert space consisting of functions $f\in L^2([0,1])$ such that $$||f||_1^2=\sum^\infty_{n=-\infty}(1+n^2)|\hat f(n)|^2<\infty;$$ where $$\hat f(n)=\frac 1 {2\pi}\int_0^1f(x)e^{-2\pi inx}{\rm d}x$$ are Fourier coefficients of $f$. Show that there exists constant $C>0$ such that $$||f||_{L^\infty}\leq C||f||_1$$ for all $f\in H_1$, where $||\cdot||_{L^\infty}$ stands for the usual supremum norm. (Hint: Use Fourier series.)

中科院李文威老师主页

通过一份李文威老师在国科大的讲义认识了他,里面有他的一些讲义。有兴趣的同学可去观摩:http://www.wwli.url.tw/index.php/zh-CN/teachingitem-zh-cn

北大本科06数分期中试题(李伟固命题)

前些年在百度文库(http://wenku.baidu.com/link?url=Y_HDeeYMcyEGuUJWt3fNqC7N08AEqMVfleNVGcv7hC2t9EVO0-MFFHWuqnLYyiDJ4H7ATgg1fOQGN1Lta2RW4Z4pOrm6aZ468WkrTqNqZzG)找到一份李伟固命制的06年北大期中考试题,当时感觉难度稍大,正好结识了Veer大神,便把这份题发给他。事实证明,V神秒得还是很顺利!

 

1.给定实数$\lambda_i(1\leq i\leq n)$,满足$\sum\limits_{i = 1}^n {\lambda _i^k} > 0\left( {k = 1,2,3, \cdots } \right)$.令$f\left( x \right) = \prod\limits_{i = 1}^n {\frac{1}{{1 - {\lambda _i}x}}}$.证明: $f^{(k)}(0)>0,k=1,2,3,\cdots$.

enlightened证明:令$$g\left( x \right) = \ln f\left( x \right) = - \ln \left( {1 - {\lambda _1}x} \right)-\ln \left( {1 - {\lambda _2}x} \right) -\cdots -\ln \left( {1 - {\lambda _n}x} \right)$$$\left( {x \in U\left( {0;\delta } \right)\text{使得对}\forall {\lambda _i},\text{有}1 - {\lambda _i}x > 0} \right)$,则

\[g\left( x \right) = \left( {\sum\limits_{i = 1}^n {{\lambda _i}} } \right)x + \left( {\frac{1}{2}\sum\limits_{i = 1}^n {\lambda _i^2} } \right){x^2} + \cdots + \left( {\frac{1}{k}\sum\limits_{i = 1}^n {\lambda _i^k} } \right){x^k} + \cdots \]由函数幂级数展开的唯一性可知${g^{\left( k \right)}}\left( 0 \right) = \frac{1}{k}\sum\limits_{i = 1}^n {\lambda _i^k} > 0\left( {x \in U\left( {0;\delta } \right)} \right)$.

 

另一方面$f\left( x \right) = {e^{g\left( x \right)}}\left( {x \in U\left( {0;\delta } \right)} \right)$.首先注意到对任意可导函数$F(x)$,有${\left( {{e^{F\left( x \right)}}} \right)^\prime } = F'\left( x \right){e^{F\left( x \right)}}$.其次注意到对可导函数组$F_1,F_2,\cdots,F_s$,有${\left( {{F_1}{F_2}{F_3} \cdots {F_s}} \right)^\prime } = {{F'}_1}{F_2}{F_3} \cdots {F_s} + {F_1}{F_2}^\prime {F_3} \cdots {F_s} + \cdots + {F_1}{F_2}{F_3} \cdots {F_s}^\prime$,从而归纳可证

\[{f^{\left( k \right)}}\left( x \right) = {\left( {{e^{g\left( x \right)}}} \right)^{\left( k \right)}} = \left( {\sum\limits_{j \in {N_ + },{k_i} \in {N_ + }} {{g^{\left( {{k_1}} \right)}}\left( x \right){g^{\left( {{k_2}} \right)}}\left( x \right) \cdots {g^{\left( {{k_j}} \right)}}\left( x \right)} } \right){e^{g\left( x \right)}}.\]由${g^{\left( k \right)}}\left( 0 \right) > 0,k=1,2,3,\cdots$且$g(0)=0$,所以${f^{\left( k \right)}}\left( 0 \right) > 0,k=1,2,3,\cdots$.cool

补充:可知\[f\left( x \right) = \prod\limits_{i = 1}^n {\frac{1}{{1 - {\lambda _i}x}}} = \left( {1 + {\lambda _1}x + \lambda _1^2{x^2} + \cdots } \right)\left( {1 + {\lambda _2}x + \lambda _2^2{x^2} + \cdots } \right) \cdots \left( {1 + {\lambda _n}x + \lambda _n^2{x^2} + \cdots } \right).\]由幂级数的乘积公式归纳可得

\[{f^{\left( k \right)}}\left( 0 \right) = \sum\limits_{\substack{{k_1} + {k_2} + \cdots + {k_n} = k\\ \left( {{k_1},{k_2}, \cdots ,{k_n}} \right)}} {\lambda _1^{{k_1}}\lambda _2^{{k_2}} \cdots \lambda _n^{{k_n}}} \left( \text{其中}{{k_i} \in N} \right).\]若能通过$\sum\limits_{i = 1}^n {\lambda _i^k} > 0\left( {k = 1,2,3, \cdots } \right)$得出$\sum\limits_{\substack{{k_1} + {k_2} + \cdots + {k_n} = k\\ \left( {{k_1},{k_2}, \cdots ,{k_n}} \right)}} {\lambda _1^{{k_1}}\lambda _2^{{k_2}} \cdots \lambda _n^{{k_n}}}>0$即可得到证明.

 

 

2.令$D = \left\{ {u = \left( {x,y} \right) \in {\mathbb{R}^2}\left| {\left\| u \right\| = \sqrt {{x^2} + {y^2}} \le \frac{1}{2}} \right.} \right\}$. $f(u)=f(x,y)$是全平面上的连续可微函数满足$\left\| {\nabla f\left( {0,0} \right)} \right\| = 1,\left\| {\nabla f\left( u \right) - \nabla f\left( v \right)} \right\| \le \left\| {u - v} \right\|$.那么对于任意的$u,v\in D$,证明函数$f|_D$在$D$中唯一点处达到其最大值.

enlightened证明:对$u\in D$,有$\left\| {\nabla f\left( u \right) - \nabla f\left( {0,0} \right)} \right\| \le \left\| {u - \left( {0,0} \right)} \right\|$,即

\[1 - \left\| {\nabla f\left( u \right)} \right\| = \left\| {\nabla f\left( {0,0} \right)} \right\| - \left\| {\nabla f\left( u \right)} \right\| \le \left\| {\nabla f\left( u \right) - \nabla f\left( {0,0} \right)} \right\| \le \left\| u \right\|,\]亦即$1 - \left\| u \right\| \le \left\| {\nabla f\left( u \right)} \right\|$,则$\nabla f\left( u \right) \ne \left( {0,0} \right)$,所以$f$的最大值只可能在边界上取得.

 

易知$f$在其边界上的函数可设为\[g\left( t \right) = f\left( {\frac{1}{2}\cos \theta ,\frac{1}{2}\sin \theta } \right),g'\left( \theta \right) = \frac{1}{2}\left( { - \sin \theta {f_x} + \cos \theta {f_y}} \right),\theta \in \left[ {0,2\pi } \right).\]现假设$f$在其边界上有两点取得最大值,不妨设为$\theta=\theta_1$和$\theta=\theta_2$,记${u_1} = \left( {\frac{1}{2}\cos {\theta _1},\frac{1}{2}\sin {\theta _1}} \right),{u_2} = \left( {\frac{1}{2}\cos {\theta _2},\frac{1}{2}\sin {\theta _2}} \right)$,则由$g'\left( {{\theta _1}} \right) = g'\left( {{\theta _2}} \right) = 0$可得$- \sin {\theta _1}{f_x}\left( {{u_1}} \right) + \cos {\theta _1}{f_y}\left( {{u_1}} \right) = 0$,即$\nabla f\left( {{u_1}} \right)$与$\left( { - \sin {\theta _1},\cos {\theta _1}} \right)$垂直即可.设$\nabla f\left( {{u_1}} \right) = {a_1}\left( {\cos {\theta _1},\sin {\theta _1}} \right)$,由于$f$在$u_1$处取得最大值,则$f$在$u_1$点沿方向$(\cos\theta_1,\sin \theta_1)$的方向导数需大于等于$0$,所以$a_1\geq 0$.

 

另一方面由$\left\| {\nabla f\left( {0,0} \right)} \right\| - \left\| {\nabla f\left( {{u_1}} \right)} \right\| \le \left\| {\nabla f\left( {0,0} \right) - \nabla f\left( {{u_1}} \right)} \right\|$当且仅当${\nabla f\left( {{0,0}} \right)}$与${\nabla f\left( {{u_1}} \right)}$异向取等可知

\[\left\| {\nabla f\left( {0,0} \right)} \right\| - \left\| {{u_1}} \right\| \le \left\| {\nabla f\left( {{u_1}} \right)} \right\| = \left| {{a_1}} \right|,\]

即$a_1\geq \frac12$.同理$\nabla f\left( {{u_2}} \right) = {a_2}\left( {\cos {\theta _2},\sin {\theta _2}} \right),a_2\geq \frac12$.由于不等式取等需要与${\nabla f\left( {0,0} \right)}$异向,故$a_1\geq\frac12,a_2\geq\frac12$中有一个是严格的.不妨设为$a_1>\frac12$,再设$\overrightarrow {{r_1}} = \left( {\cos {\theta _1},\sin {\theta _1}} \right),\overrightarrow {{r_2}} = \left( {\cos {\theta _2},\sin {\theta _2}} \right)$,则

 

\begin{align*}&{\left\| {\nabla f\left( {{u_1}} \right) - \nabla f\left( {{u_2}} \right)} \right\|^2} - {\left\| {{u_1} - {u_2}} \right\|^2}\\= &{\left\| {{a_1}\overrightarrow {{r_1}} - {a_2}\overrightarrow {{r_2}} } \right\|^2} - {\left\| {\frac{1}{2}\overrightarrow {{r_1}} - \frac{1}{2}\overrightarrow {{r_2}} } \right\|^2} = a_1^2 + a_2^2 - 2{a_1}{a_2}\overrightarrow {{r_1}} \overrightarrow {{r_2}} - \left( {\frac{1}{4} - \frac{1}{2}\overrightarrow {{r_1}} \overrightarrow {{r_2}} + \frac{1}{4}} \right)\\= &a_1^2 + a_2^2 - \frac{1}{2} - \left( {2{a_1}{a_2} - \frac{1}{2}} \right)\overrightarrow {{r_1}} \overrightarrow {{r_2}} \left( \text{由于}{\overrightarrow {{r_1}}\text{与} \overrightarrow {{r_2}} \text{不同向},\text{所以}\overrightarrow {{r_1}} \overrightarrow {{r_2}} < 1,\text{且}2{a_1}{a_2} - \frac{1}{2} > 0} \right)\\> &a_1^2 + a_2^2 - \frac{1}{2} - \left( {2{a_1}{a_2} - \frac{1}{2}} \right) = {\left( {{a_1} - {a_2}} \right)^2} \ge 0,\end{align*}

 

因此${\left\| {\nabla f\left( {{u_1}} \right) - \nabla f\left( {{u_2}} \right)} \right\|^2} - {\left\| {{u_1} - {u_2}} \right\|^2} > 0$,即$\left\| {\nabla f\left( {{u_1}} \right) - \nabla f\left( {{u_2}} \right)} \right\| > \left\| {{u_1} - {u_2}} \right\|$,矛盾,从而$f$在边界上只有一点取得最大值,即函数$f|_D$在$D$中唯一点处达到其最大值.cool

 

3.讨论级数$\sum\limits_{n = 1}^{ + \infty } {{{\left( { - 1} \right)}^{n - 1}}\frac{{\sin \left( {\ln n} \right)}}{{{n^\alpha }}}},\alpha\in \mathbb{R}$的收敛性.

 

enlightened解:(1)当$\alpha\leq0$时,我们可知$\mathop {\lim }\limits_{n \to \infty } {\left( { - 1} \right)^{n - 1}}\frac{{\sin \left( {\ln n} \right)}}{{{n^\alpha }}}$发散,从而级数$\sum\limits_{n = 1}^{ + \infty } {{{\left( { - 1} \right)}^{n - 1}}\frac{{\sin \left( {\ln n} \right)}}{{{n^\alpha }}}}$发散.

 

(2)当$\alpha>0$时,由于$\mathop {\lim }\limits_{n \to \infty } {\left( { - 1} \right)^{n - 1}}\frac{{\sin \left( {\ln n} \right)}}{{{n^\alpha }}} = 0$,我们讨论其前$2n$项和数列的收敛性即可,也就是级数$\sum\limits_{n = 1}^{ + \infty } {\left[ {\frac{{\sin \left( {\ln \left( {2k - 1} \right)} \right)}}{{{{\left( {2k - 1} \right)}^\alpha }}} - \frac{{\sin \left( {\ln \left( {2k} \right)} \right)}}{{{{\left( {2k} \right)}^\alpha }}}} \right]}$的收敛性.

 

\begin{align*}&\left| {\frac{{\sin \left( {\ln \left( {2k - 1} \right)} \right)}}{{{{\left( {2k - 1} \right)}^\alpha }}} - \frac{{\sin \left( {\ln \left( {2k} \right)} \right)}}{{{{\left( {2k} \right)}^\alpha }}}} \right|\\= &\left| {\sin \left( {\ln \left( {2k - 1} \right)} \right)\left( {\frac{1}{{{{\left( {2k - 1} \right)}^\alpha }}} - \frac{1}{{{{\left( {2k} \right)}^\alpha }}}} \right) + \frac{{\sin \left( {\ln \left( {2k - 1} \right)} \right) - \sin \left( {\ln \left( {2k} \right)} \right)}}{{{{\left( {2k} \right)}^\alpha }}}} \right|\\\le &\left| {\sin \left( {\ln \left( {2k - 1} \right)} \right)} \right|\left[ {\frac{1}{{{{\left( {2k - 1} \right)}^\alpha }}} - \frac{1}{{{{\left( {2k} \right)}^\alpha }}}} \right] + \left| {\frac{{2\cos \left[ {\frac{{\ln \left( {2k - 1} \right) + \ln \left( {2k} \right)}}{2}} \right]\sin \left[ {\frac{{\ln \left( {2k - 1} \right) - \ln \left( {2k} \right)}}{2}} \right]}}{{{{\left( {2k} \right)}^\alpha }}}} \right|\\\le &\left[ {\frac{1}{{{{\left( {2k - 1} \right)}^\alpha }}} - \frac{1}{{{{\left( {2k} \right)}^\alpha }}}} \right] + \left| {\ln \left( {1 - \frac{1}{{2k}}} \right)} \right|\frac{1}{{{{\left( {2k} \right)}^\alpha}}}.\end{align*}

已知$\left| {\ln \left( {1 - \frac{1}{{2k}}} \right)} \right|\frac{1}{{{{\left( {2k} \right)}^\alpha }}} \sim \frac{1}{{{{\left( {2k} \right)}^{\alpha + 1}}}}$,从而$\sum\limits_{k = 1}^{ + \infty } {\left| {\ln \left( {1 - \frac{1}{{2k}}} \right)} \right|\frac{1}{{{{\left( {2k} \right)}^\alpha }}}}$收敛.又$\sum\limits_{k = 1}^{ + \infty } {\left[ {\frac{1}{{{{\left( {2k - 1} \right)}^\alpha }}} - \frac{1}{{{{\left( {2k} \right)}^\alpha }}}} \right]}$是收敛的,故$\sum\limits_{n = 1}^{ + \infty } {\left| {\frac{{\sin \left( {\ln \left( {2k - 1} \right)} \right)}}{{{{\left( {2k - 1} \right)}^\alpha }}} - \frac{{\sin \left( {\ln \left( {2k} \right)} \right)}}{{{{\left( {2k} \right)}^\alpha }}}} \right|}$收敛,因此$\sum\limits_{n = 1}^{ + \infty } {\left[ {\frac{{\sin \left( {\ln \left( {2k - 1} \right)} \right)}}{{{{\left( {2k - 1} \right)}^\alpha }}} - \frac{{\sin \left( {\ln \left( {2k} \right)} \right)}}{{{{\left( {2k} \right)}^\alpha }}}} \right]}$亦收敛.

综上所述, 当$\alpha\leq 0$时,级数发散;当$\alpha>0$时级数收敛.cool

 

3.求\[\mathop {\inf }\limits_{n \ge 1} \left\{ {\mathop {\min }\limits_{x \in \left[ {0,\frac{\pi }{2}} \right]} \sum\limits_{k = 1}^n {\frac{{\cos kx}}{k}} } \right\}.\]

enlightened解:首先, $\mathop {\min }\limits_{x \in \left[ {0,\frac{\pi }{2}} \right]} \sum\limits_{k = 1}^1 {\frac{{\cos kx}}{k}} = \mathop {\min }\limits_{x \in \left[ {0,\frac{\pi }{2}} \right]} \cos x = 0,\mathop {\min }\limits_{x \in \left[ {0,\frac{\pi }{2}} \right]} \sum\limits_{k = 1}^2 {\frac{{\cos kx}}{k}} = \mathop {\min }\limits_{x \in \left[ {0,\frac{\pi }{2}} \right]} \left( {\cos x + \frac{{\cos 2x}}{2}} \right) = - \frac{1}{2}$.

 

现记${f_n}\left( x \right) = \sum\limits_{k = 1}^n {\frac{{\cos kx}}{k}}$,则$\mathop {\min }\limits_{x \in \left[ {0,\frac{\pi }{2}} \right]} {f_1}\left( x \right) = 0 \ge - \frac{1}{2},\mathop {\min }\limits_{x \in \left[ {0,\frac{\pi }{2}} \right]} {f_1}\left( x \right) = - \frac{1}{2} \ge - \frac{1}{2}$.

 

现假设$n=k-1(k\geq2,k\in\mathbb{N}_+)$时,有$\mathop {\min }\limits_{x \in \left[ {0,\frac{\pi }{2}} \right]} {f_n}\left( x \right) = \mathop {\min }\limits_{x \in \left[ {0,\frac{\pi }{2}} \right]} {f_{k - 1}}\left( x \right) \ge - \frac{1}{2}$成立;

 

当$n=k$时,注意到\[{f_n}^\prime \left( x \right) = \sum\limits_{k = 1}^n {\left( { - \sin kx} \right)} = \sum\limits_{i = 1}^k {\left( { - \sin ix} \right)} = \sum\limits_{i = 1}^k {\frac{{\cos \left( {i + \frac{1}{2}} \right)x - \cos \left( {i - \frac{1}{2}} \right)x}}{{2\sin \frac{x}{2}}}} ,\]

则\[{f_k}^\prime \left( x \right) = \frac{{\cos \left( {k + \frac{1}{2}} \right)x - \cos \frac{x}{2}}}{{2\sin \frac{x}{2}}} = - \frac{{\sin \frac{{k + 1}}{2}x\sin \frac{k}{2}x}}{{\sin \frac{x}{2}}},x \in \left[ {0,\frac{\pi }{2}} \right].\]

由$f_k(x)$的连续性可知$\min f_k(x)$的点只可能在端点或稳定点,从而令$f'_k(x)=0$,则$x = \frac{{2j\pi }}{{k + 1}},\frac{{2j\pi }}{k},j \in \mathbb{Z}$且$\frac{{2j\pi }}{{k + 1}},\frac{{2j\pi }}{k} \in \left( {0,\frac{\pi }{2}} \right)$,而

\[{f_k}\left( {\frac{{2j\pi }}{{k + 1}}} \right) = \sum\limits_{i = 1}^n {\frac{{\cos i\left( {\frac{{2j\pi }}{{k + 1}}} \right)}}{i}} = {f_{k - 1}}\left( {\frac{{2j\pi }}{{k + 1}}} \right) + \frac{{\cos \frac{{2kj\pi }}{{k + 1}}}}{k} = {f_{k - 1}}\left( {\frac{{2j\pi }}{{k + 1}}} \right) + \frac{{\cos \frac{{2j\pi }}{{k + 1}}}}{k}.\]

由于$\frac{{2j\pi }}{{k + 1}} \in \left[ {0,\frac{\pi }{2}} \right]$,所以$\frac{{\cos \frac{{2j\pi }}{{k + 1}}}}{k} > 0$,则\[{f_k}\left( {\frac{{2j\pi }}{{k + 1}}} \right) = {f_{k - 1}}\left( {\frac{{2j\pi }}{{k + 1}}} \right) + \frac{{\cos \frac{{2j\pi }}{{k + 1}}}}{k} \ge - \frac{1}{2}.\]

而${f_k}\left( {\frac{{2j\pi }}{k}} \right) = \sum\limits_{i = 1}^n {\frac{{\cos i\left( {\frac{{2j\pi }}{k}} \right)}}{i}} = {f_{k - 1}}\left( {\frac{{2j\pi }}{k}} \right) + \frac{1}{k} \ge - \frac{1}{2}$,又

${f_k}\left( 0 \right) = \sum\limits_{i = 1}^k {\frac{1}{i}} > - \frac{1}{2},{f_k}\left( {\frac{\pi }{2}} \right) = \sum\limits_{i = 1}^k {\frac{{\cos i\frac{\pi }{2}}}{i}} = \left\{ \begin{array}{l}\frac{1}{2}\left[ { - 1 + \left({\frac{1}{2} - \frac{1}{3}} \right) + \cdots + \left( {\frac{1}{{s - 1}} - \frac{1}{s}} \right)} \right],k = 2s + 1\\\frac{1}{2}\left[ { - 1 + \left( {\frac{1}{2} - \frac{1}{3}} \right) + \cdots + \left( {\frac{1}{{s - 1}} - \frac{1}{s}} \right)} \right],k = 2s\end{array} \right.$,从而${f_k}\left( {\frac{\pi }{2}} \right) \ge - \frac{1}{2}$.

 

综上归纳可知$\mathop {\min }\limits_{x \in \left[ {0,\frac{\pi }{2}} \right]} {f_n}\left( x \right) = \mathop {\min }\limits_{x \in \left[ {0,\frac{\pi }{2}} \right]} \sum\limits_{k = 1}^n {\frac{{\cos kx}}{k}} \ge - \frac{1}{2}$,又$\mathop {\min }\limits_{x \in \left[ {0,\frac{\pi }{2}} \right]} \sum\limits_{k = 1}^2 {\frac{{\cos kx}}{k}} = - \frac{1}{2}$,因此

\[\mathop {\inf }\limits_{n \ge 1} \left\{ {\mathop {\min }\limits_{x \in \left[ {0,\frac{\pi }{2}} \right]} \sum\limits_{k = 1}^n {\frac{{\cos kx}}{k}} } \right\} = - \frac{1}{2}.\]

cool

 

4.函数$f(x)$在$[0,1]$上二次可导, $f(0)=2,f'(0)=-2,f(1)=1$.证明存在$c\in (0,1)$,使得$f(c)f'(c)+f''(c)=0$.

enlightened证明:令$F\left( x \right) = \frac{1}{2}{f^2}\left( x \right) + f'\left( x \right)$,则$F(0)=\frac12\times 2^2-2=0$.现假设不存在$\xi\in (0,1]$使得$F(\xi)=0$,则$F$在$(0,1]$上不变号.倘若$\forall x\in (0,1]$都有$F(x)<0$,即$\frac{1}{2}{f^2}\left( x \right) + f'\left( x \right) < 0$,则$f'\left( x \right) < - \frac{1}{2}{f^2}\left( x \right) \le 0$,则$f$在$[0,1]$上单调递减,由$f(0)=2,f(1)=1$可知$1\leq f(x)\leq 2,f(x)\neq 0$,从而$\frac{{f'\left( x \right)}}{{{f^2}\left( x \right)}} < - \frac{1}{2}$.由$f,f'$的连续性可知$\int_0^1 {\frac{{f'\left( x \right)}}{{{f^2}\left( x \right)}}dx} < \int_0^1 {\left( { - \frac{1}{2}} \right)dx}$,即$\left. { - \frac{1}{{f\left( x \right)}}} \right|_0^1 < - \frac{1}{2}$,得到$-\frac12<-frac12$,矛盾.

 

倘若$\forall x\in (0,1]$都有$F(x)>0$,即$\frac{1}{2}{f^2}\left( x \right) + f'\left( x \right) > 0$,由于$f(1)=1\neq 0$,则$\forall x\in U^-(1,\delta)$,有$\int_x^1 {\frac{{f'\left( t \right)}}{{{f^2}\left( t \right)1}}dt} > \int_x^1 {\left( { - \frac{1}{2}} \right)dt}$,即$\left. { - \frac{1}{{f\left( t \right)}}} \right|_x^1 > \frac{{x - 1}}{2}$,从而$\frac{1}{{f\left( x \right)}} > \frac{{x + 1}}{2}$.

 

当$x\in U^-(1,\delta)$时,有$\frac{1}{{f\left( x \right)}} > \frac{{x + 1}}{2} > 0\left( {\delta \le 1} \right)$,故$f(x)$在$[0,1]$上不为$0$,则$\frac{1}{{f\left( 0 \right)}} > \frac{{0 + 1}}{2}$,则$\frac12>\frac12$,矛盾.

 

因此存在$\xi\in (0,1]$使得$F(\xi)=0$,从而$F(0)=F(\xi)=0$,由罗尔定理可知$\exists c\in (0,1)$,使得

\[F'\left( c \right) = f\left( c \right)f'\left( c \right) + f''\left( c \right) = 0.\]

cool

 

5.$A$和$B$是自然数$\mathbb{N}$的两个无穷子集,满足$A\cap B=\text{空集},A\cup B=\mathbb{N}$,对于任意的自然数$c>0$,是否存在两个递增的数列$\{a_n\},\{b_n\},\{a_n\}\in A,\{b_n\}\in B$,使得$\mathop {\lim }\limits_{n \to \infty } \frac{{{a_n}}}{{{b_n}}} = c$.

enlightened证明:对$\forall l\geq s\in \mathbb{N}_+$.记$s\sim l=\{s,s+1,\cdots,l\}$,则若对$\forall N\in \mathbb{N}_+$, $\exists n_2\geq n_1\geq N\geq c^2$有$x_{n_1}\sim x_{n_2}\subset B$.令$x_{n_1}=v_1,x_{n_2}=u_1$,则$v_1\sim u_1\subset B$.若$cv_1\sim cu_1+c[\sqrt{u_1}+1]\not\subset B$,则$\exists a\in A,b\in B$,使得$|a-cb|\leq c$或$|a-cb|\leq \sqrt{b}+1$.

 

若$cv_1\sim cu_1+c[\sqrt{u_1}+1]\subset B$,令$u_2=u_1+[\sqrt{u_1}+1]$,则$cv_1\sim cu_2\subset B$.以此类推,若始终成立$c^kv_1 \sim c^k u_{k+1}\subset B$,易知$u_{k+1}=u_k+[\sqrt{u_k}+1]\geq u_k+\sqrt{u_1}$,即$u_{k+1}-u_k\geq \sqrt{u_1}$,所以$u_k$有趋于无穷大的趋势,所以存在某个$k_0\in \mathbb{N}_+$,使得$\frac{u_{k_0}}{v_1}>c$.而$c^{k_0}v_1\sim c^{k_0} u_{k_0+1}\subset B$.令$x_{n_{k_0}}\in B$满足$c^{k_0}v_1\sim c^{k_0}u_{k_0+1}\subset c^{k_0}v_1\sim x_{n_{k_0}}\subset B,x_{n_{k_0}}+1\in A$,令$a_1=x_{n_{k_0}}+1$,则$c^{k_0}v_1<\frac{a_1}{c}\leq x_{n_{k_0}+1}$,则$\exists b_1\in B$,使得$\left| {\frac{{{a_1}}}{c} - {b_1}} \right| \le 1$,即$\left| {{a_1} - c{b_1}} \right| \le c$.

 

若不始终成立$c^kv_1\sim c^ku_{k+1}$,则$\exists b_1\in B,a_1\in A$有$|a_1-cb_1|\leq c$或$|a_1-cb_1|\leq \sqrt{b_1}+1$,即$|a_1-cb_1|\leq \sqrt{a_1}+1$.接着再取$n_1\geq n_2\geq N$,有$x_{n_2}\geq x_{n_1}>a_1,b_1$且$x_{n_1}\sim x_{n_2}\subset B$,则$\exists a_2\in A,b_2\in B$有$|a_2-cb_2|\leq \sqrt{b_2}+1,\cdots$,故存在递增数列$\{a_n\}\subset A,\{b_n\}\subset B$,有$|a_n-cb_n|\leq \sqrt{b_n}+1$,则$\left| {\frac{{{a_n}}}{{{b_n}}} - c} \right| \le \frac{1}{{\sqrt {{b_n}} }} + \frac{1}{{{b_n}}}$,所以$\mathop {\lim }\limits_{n \to \infty } \frac{{{a_n}}}{{{b_n}}} = c$.cool

碰到的数学分析中的反例

  • 在$\mathbb{R}$上连续的有界函数一定一致连续吗?
kiss考察函数$$f(x)=\sin x^2.$$
  • 设函数$f(x)$在区间$[a,b]$上可微,则$f'(x)$在$[a,b]$上有界?
kiss考察函数\[f(x)=x^{\frac32}\sin\frac1x.\]
  • 存不存在这样函数, $f(a)=0$, $f(x)$在区间$[a,b]$上连续,在区间$(a,b)$上可导且$f(x)>0$,而对任意$\varepsilon>0$,函数在区间$(a,a+\varepsilon)$是不单调递增的?
kiss考察函数\[x\sin \frac1x+10x.\]
  • 是否存在仅在一点可导而在该点之外的每一点都不可导的函数?
kiss考察函数\[f\left( x \right) = \left\{ \begin{array}{l}{x^2},x\text{为无理数}\\0,x\text{为有理数}\end{array} \right.\]在点$x=0$即可.